Trouvez des réponses facilement sur Laurentvidal.fr, la plateforme de Q&R de confiance. Connectez-vous avec une communauté d'experts prêts à vous aider à trouver des solutions précises à vos interrogations de manière rapide et efficace. Trouvez des solutions détaillées à vos questions grâce à une large gamme d'experts sur notre plateforme conviviale de questions-réponses.

Maths - term voire supérieur
Bonjour, est-ce que quelqu'un pourrait m'aider à prouver que la formule de Madhava-Leibniz converge vers π ? Ladite formule :
Merci !

Maths Term Voire Supérieur Bonjour Estce Que Quelquun Pourrait Maider À Prouver Que La Formule De MadhavaLeibniz Converge Vers Π Ladite Formule Merci class=

Sagot :

Explications étape par étape:

Bonsoir, alors, cette démonstration est très complexe ! J'ignore pourquoi tu la veux, elle ne semble pas exister sur internet en tout cas.

L'idée étant de prouver que la somme infinie converge et tend vers π / 4, ainsi, en la multipliant par 4, la formule de Madhava-Leibniz sera validée. Ce sera très long, prends un café, un casque, ce que tu veux.

En premier lieu, il faut déjà prouver la convergence de cette somme infinie. Étant donné qu'il s'agit d'une série à termes quelconques, pas le choix, on doit passer par des séries adjacentes, une paire, l'autre impaire. La convergence absolue ne fonctionne pas ici.

Notons Sn cette série, pour k allant de 0 à n.

Considérons 2 suites extraites, Rn = S(2n) et Tn = S(2n+1).

Étudions désormais le monotonie de Rn :

R(n+1) - Rn = S(2n+2) - S(2n) = [(-1)^(2n+2) / (4n+5)] - [(-1)^(2n) / (4n+1)]

= - [(-1)^(2n+1) / (4n+5)] + [(-1)^(2n+1) / (4n+1)]

= (-1)^(2n+1) * [1 / (4n+1) - 1 / (4n+5)]

= - 1 / (4n+1) + 1 / (4n+5) (car 2n+1 impair).

= - 4 / ((4n+1)(4n+5)) > 0.

La suite Rn est donc décroissante pour tout entier naturel n.

Même procédé pour Tn, T(n+1) - Tn = S(2n+3) - S(2n+1) = [(-1)^(2n+3) / (4n+7)] - [(-1)^(2n+1) / (4n+3)]

= - [(-1)^(2n+2) / (4n+7)] + [(-1)^(2n+2) / (4n+3)]

= 1 / (4n+3) - 1 / (4n+7) (car 2n+2 pair).

= 4 / (4n+3)(4n+7) > 0, donc Rn croissante.

En effectuant la différence Tn - Rn, on peut prédire qu'il y aura au numérateur un polynôme en n du 2e degré, et au dénominateur, un polynôme en n du 4e degré. On factorisera par le terme de plus haut degré au numérateur ainsi qu'au dénominateur, la limite tendra donc, par quotient, vers 0.

Tn et Rn sont donc adjacentes, ce qui prouve leur convergence, elles admettent aussi la même limite. Ceci suffit à prouver la convergence de Sn. Il ne suffira plus qu'à faire tendre n vers l'infini, la somme infinie sera donc convergente.

1re partie terminée, désormais, il faut trouver cette limite !

Ici, il faut être astucieux. Posons f(x) = arctan(x), qui est infiniment dérivable sur [0 ; 1].

Maintenant, il nous faut utiliser la dérivée k-ieme de arctan x, on peut la trouver sur internet (les calculs seront monstrueux sinon).

Pour simplifier, car on sait d'avance que f(x) = Sn(x) (formule de Taylor en 0), il suffit de poser x = 1, pour obtenir la somme infinie.

Ne pas oublier que : arctan(x) = x - (x^3 / 3) + (x^5 / 5) +...+ (-1)^k * (x^(2n+1)) / (2n+1) + o(2n+1).

Avec l'inégalité de Taylor-Lagrange, on prouverait que | arctan(1) - Sn(1) | tend vers 0 lorsque n tend vers + infini.

Ainsi, la limite de la somme infinie vaut arctan(1) = pi/4, ce qui est la valeur recherchée.

Fin de la démonstration (je te laisse la 2e partie, si tu te sens le courage de majorer la dérivée n-ieme de arctan x, t'auras une somme avec des k et n factorielle à majorer aussi).

Je te déconseille aussi d'utiliser la formule de Stirling, elle ne t'aidera pas + que ça :p

Bonne soirée !

Nous espérons que cela vous a été utile. Revenez quand vous voulez pour obtenir plus d'informations ou des réponses à vos questions. Merci d'avoir choisi notre plateforme. Nous nous engageons à fournir les meilleures réponses à toutes vos questions. Revenez nous voir. Nous sommes ravis de répondre à vos questions sur Laurentvidal.fr. N'oubliez pas de revenir pour en savoir plus.